Orthogonality of Cosine and Sine Functions: A Proof by Explicit Integration

  • Thread starter Thread starter Jack21222
  • Start date Start date
  • Tags Tags
    Cos Sin
Click For Summary
The discussion centers on proving that the integral from 0 to a of sin(m*pi*x/a)*cos(n*pi*x/a) equals zero through explicit integration. The user transformed the product into a sum of sine functions and found that the integral yields zero when m+n is even, but a non-zero result when m+n is odd. They verified their calculations using a TI-89 calculator and Mathematica, which confirmed their findings. The user questioned the original problem's formulation, suspecting a mistake in the arguments of the sine and cosine functions. The professor later confirmed that the correct formulation should include a factor of 2 in the arguments, resolving the confusion.
Jack21222
Messages
209
Reaction score
1

Homework Statement



This would be easier if I knew how to use LaTeX, but I'll give it a shot.

Show by explicit integration that the following is true:

Integral from 0 to a of sin(m*pi*x/a)*cos(n*pi*x/a)=0

Homework Equations


The Attempt at a Solution



I've used an identity to change sin(m*pi*x/a)*cos(n*pi*x/a) into sin[(m+n)*pi*x/a]+sin[(m-n)*pi*x/a] and integrated each side. When I do this, m+n=even (including m=n), the answer is zero, but the answer is not zero for m+n=odd. Instead, I get -a*m/[(m^2-n^2)*pi]

So, I decided to check with my TI-89. When I enter the original problem into my TI-89, I get the same thing.

Mathematica tells me it equals -((a (-m + m Cos[m \[Pi]] Cos[n \[Pi]] +
n Sin[m \[Pi]] Sin[n \[Pi]]))/((m^2 - n^2) \[Pi])), which ends up reducing to the same thing. I tested Mathematica with m = 2 and n = 3, and it gave me -4a/(5*pi), which is the same as my TI-89 and the same as the answer I got by hand.

Is the question wrong?
 
Last edited:
Physics news on Phys.org
Are the arguments of the functions really \frac{m \pi x}{a}?

Perhaps it should say \frac{m 2 \pi x}{a}
 
Stephen Tashi said:
Are the arguments of the functions really \frac{m \pi x}{a}?

Perhaps it should say \frac{m 2 \pi x}{a}

I emailed the professor after posting this, and he came back with the correction that yes, it should have been 2*pi, not pi. He forgot the factor of 2.

Everything's correct now, thanks. If I had hair, I would have pulled it out over this. :-p
 
Question: A clock's minute hand has length 4 and its hour hand has length 3. What is the distance between the tips at the moment when it is increasing most rapidly?(Putnam Exam Question) Answer: Making assumption that both the hands moves at constant angular velocities, the answer is ## \sqrt{7} .## But don't you think this assumption is somewhat doubtful and wrong?

Similar threads

Replies
1
Views
1K
  • · Replies 6 ·
Replies
6
Views
2K
  • · Replies 14 ·
Replies
14
Views
2K
  • · Replies 5 ·
Replies
5
Views
2K
Replies
3
Views
2K
Replies
3
Views
2K
  • · Replies 5 ·
Replies
5
Views
2K
Replies
3
Views
2K
  • · Replies 9 ·
Replies
9
Views
1K
Replies
2
Views
1K